Sei sulla pagina 1di 76
Perera | HANDBOOK TO-BS-5628: . - — PART 2: SECTION 2: ° ee Reinforced Masonry Design aad SYMBOLS LIST OF FIGURES 10 INTRODUCTION 2.0 CODECLAUSES General Code Section 3 Clause 16, Basis of design Stability ral properties and anal compressive strength of mason Characteristic compressive strength of masonry in bending Characteristic shear strength of masonry, f Characteristic strength of reinforcing stel,f, Characteristic anchorage bond stren; Elastic moduli, Ey Ex, Es Analysis of structure ‘Clause 20, Partial safety factors Code Section 4 Clause 21, General coer re “ Resistance moment of elements . Re ate A Clause 23, Reinforced masonry subjected to a combination of vergfcal loading and bending RV ECer a cet L aun ee rere Curd 2.3.5 Clause 25, Reinforced masonry subjected to horizontal forces ingghe plane of the element 23.6 Clause 26, Detailing reinforced masonry Peas ep a resist) cement Dee oe eto ead Re ee PRM aCe el hua) ay ee meen ‘Technical editor: Peter Watt BSc (Eng) REINFORCED MASONRY DESIGN EXAMPLES Beams Example 1, Simply supported beam Example 2, Cantilever beam Columns Example 3, Axially loaded short column Example 4, Axially loaded slender colun Example 5, Short column and slender column subjected to single axis bending Example 6, Short column subjected to biaxial bending Retaining walls Example 7, Grouted cavity retaining wall Example 8, Quetta bond retaining wall Example 9, Pocket retaining Deen re ea Rn Soe bee Fouad eae eae ee BS 5628: Part 2, Appendix A Design Methods ORV Tots eRe hard POR eed ee od REFERENCES oes pt EEC SELL Sed yaaeb ey ‘Common bond patterns for reinforced masonry 7 Front cover: Dormitory block at the American Community School in Surrey uses reinforced brickwork. This page: Reinforced brickwork subway at Banbury uses pocket type" retaining walls. Handbook to BSS628: Part2: SECTION? SYMBOLS Ky PE%SEEEEEEEOO PY D2 OY VEZ PR 90 Zz cross-sectional area of masonry cross-sectional area of masonry in compression cross-sectional area of primary reinforcing steel area of compression reinforcement in the most compressed face area of reinforcement in the least compressed face cross-sectional area of reinforcing steel resisting shear forces shearspan distance from face of support to the nearest edge of a principal load width of section width of compression face midway between restraints width of rib orpier width of section at level ofthe tension reinforcement lever arm factor effective depth depth of masonry in compression the depth from the surface to the reinforcement in the more highly compressed face the depth of the centroid of the reinforcement from the least compressed face modulus of elasticity of concrete modulus of elasticity of masonry nominal earth or water load modulus of elasticity of stee! resultant eccentricity in plane of bending characteristic anchorage bond strength between mortar or concrete infill and steel characteristic compressive strength of masonry characteristic flexural strength (tension) of masonry stressin the reinforcement stressin the reinforcement in the most compressed face stress in the reinforcement in the least compressed face characteristic shear strength of masonry characteristic tensile strength of reinforcing stcel characteristic dead load acceleration due to gravity design load per unit area due to loads acting at right angles to the bed joints clear distance between lateral supports effective height of wall or column moment of inertia active ground pressure coefficient length of the wall span of beam bending moment due to design load inerease in moment due to slenderness design moment of resistance design moment about the x axis effective uniaxial design moment about the x axis design moment about the y axis effective uniaxial design moment about the y axis ratio of E, tO Em design axial load design axial loud resistance design axial load resistance of column, lateral force due to earth pressure overall section dimension ina direction perpendicular to the x axis moment of resistance factor characteristic imposed load overall section dimension ina direction perpendicular to the y axis reaction arch rise spacing of reinforcing bars spacing of shear reinforcement along member overall thickness of a wall or column width of cavity effective thickness of a wall or column snoring all bending ty thickness of a lange in a pocket-type wall V_ shear force due to design loads vy shearstress due to design loads W point load Wy characteristic wind load w—_uniformly distributed load section modulus 2 ever arm @ coefficient for column design or bending moment coefficient for lateral load design ‘partial safety factor for load Ym partial safety factor for material Yn Partial safety factor for bond strength between mortar or concrete infill and steet Ymm Partial safety factor for compressive strength of masonry Ys Partial safety factor for strength ofsteel Ye Partial safety factor for shear strength of masonry Pp Abd p —_soildensity 3 angle of friction between retained earth and back of wall orthogonal ratio LIST OF FIGURES Beams and slabs— limiting deflections Overall stability ~ minimum horizontal loading Cracking due to differential movement Design width of masonry with raked joints Damp proof course in reinforced masonry 6. Characteristic compressive strength, f,, for masonry of units with ratio of height to least horizontal dimension of 0.6 7. Characteristic compressive strength, f,. for masonry of solid blocks with ratio of height to least horizontal dimension of 1.0 8. Characteristic compressive strength, f,. for masonry of solid blocks with ratio of height to least horizontal dimension of between 2.0and 4.0 9. Characteristic compressive strength, f,, for masonry of blocks other than solid with ratio of height to least horizontal dimension of between 2.0.and 4.0 10. Definition of principal load 11. Alternative cross sections for derivation of relative stiffness 12. Load cases 13. Determination of effective span 14, Limiting spans for lateral stability 15. Wall beam/tied arch analogyisimply supported beam 16. Flange thicknesses of pocket type walls 17. Flange width of pocket type walls 18. Effective width, reinforced hollow blockwork 19. Stress block-short column minimum reinforcement case 20. Stress block-short column design reinforcement case 21. Variation of depth of compression block, d., with stress in reinforcement in least compressed face of column, fs 22. Maximum reinforcement sizes 23. Pocket wall maximum reinforcement size 24. Bar spacing recommendations 25. Link anchorage 26. Arrangement of column links 27. Effective anchorage of hooks and bends 28. Reinforcement anchorage. internal supports 29. Reinforcement curtailment 30. Reinforcement anchorage, simply supported ends 31. Anchorage of reinforcement where principal loads occur 32. Typical flexural design flow chart 33, Links in vertical joints. Handbook to BSS628: Part 2: SECTION 2: Reinforced Masonry 3 The enclosing walls of this arts centre at High Wycombe are of post-tensioned calcium silicate diaphragm walling. Brickwork construction independent of the structural steelwork. ri INTRODUCTION ‘This Section of the Handbook deals with the objectives of the design of reinforced masonry to 'B$5628': Part 2 and the general and detailed design recommendations as contained in Sections 3and 4 of the Code. The intention is that, whilst reference may be made to Section 1 of this Handbook as necessary, Section 2 covers those parts of the Code that relate to the carrying out of structural design calculations. CODE CLAUSES 21 22 2.21 GENERAL Sections 3 and 4 of BS5628 Part 2 contain the information required to enable the designer of reinforced masonry to carry out structural calculations and also enable the design philosophy adopted by the Code to be followed. As already described in Section 1 of this Handbook, the Code adopts the Limit State Philosophy which has been used by many countries and organisations for their most recent Codes of Practice or equivalent design documents. Code Section 3 sets out the design objectives ‘embodied in the limit state philosophy and discusses the general stability requirements of reinforced masonry structures. Detailed design information and guidance is given in Code Section 4, Also given in Code Section 4 are recommendations on the detailing of reinforced masonry. CODE SECTION 3 CLAUSE 16, BASIS OF DESIGN ‘This clause deals with the ‘basis of design’ and outlines the requirements of the limit state approach to design as adopted by the Code. The clause also refers o the need to provide an adequate factor of safety against the ultimate limit state, ie collapse, being reached. Thus the design strength must be equal to, or exceed, moments etc, resulting from the design load. In general terms: “Ym X characteristic strength of reinforced masonry must be greater than or equal to ‘yxcharacteristic applied load effects ‘where yq and y¢are the partial safety factors on materials and loads respectively appropriate to the ultimate limit state. ‘The design must also satisfy the requirements of the serviceability limit states of deflection and cracking and/or others where appropriate. The same relationship as given above must be satisfied for each of the serviceability limit states. To satisfy this relationship partial safety factors are used, appropriate to the limit state under consideration, as given in the Code, or otherwise derived. ‘The Code suggests that the designer should consider whether the proportion of concrete infill to the overall cross-section makes reinforced concrete design to BS8110° more appropriate than BS5628 : Part 2 design, ie will the section behave as reinforced masonry or as reinforced concrete incorporating masonry. No guidance i given, however, as to what the limiting proportions should be, but it might be assumed that if more than 50% of the section is concrete, BS8110 would be more appropriate. Inrelation to the ultimate limit state the Code refers to the strength of the structure being adequate for the design loads, taking account of the possibility of overturning and buckling, ‘Thus the designer must ensure that an ultimate limit state is not reached as a result of buckling duc to elastic or plastic instability, considering, where appropriate, the effects of sway. These recommendations are not peculiar to reinforced masonry and designers experienced in the design of reinforced concrete to BS8110 will have no difficulty in coping with them. ‘Compliance with the Code requirements for the serviceability limit state of deflection is, intended to ensure that the performance of the structure or its applied finishes in service is not impaired, especially with respect to weather resistance. To this end the following limiting deflections are recommended in the Code and illustrated in Figure 1: a) Final deflection (including that due to temperature, creep, shrinkage, etc,) not greater length span than PM for cantilevers and SP2? for other elements. b) Post-construction deflecti 3 (that du to imposed oad) not greater than B22 or 20mm, whichever is less, in order to minimise risk of damage to partitions and floor finishes. [aio ns: re SCHON: Roloson 3 ‘Suecessful and economic use of post-tensioned brickwork inthis unique building for the Salvation ‘Army. The Orsborn Memorial Halls, Bournemouth «) Upward deflection of prestressed masonry members not greater than: Stay where finishes are tobe applied unless uniformity ofamberbetween adjacent nits canbe ensured ‘These recommendations were based on those which were given in CP110° with the exception of the additonal imitation oncantlevermembersgivenin) andthe eduction n permitted defection inb) from $22 to P2P. These reflect the conservative approach ofthe Code to actual performance of enforced masonry in areain which nformationislimited Item c) above applies to factory prestressed masonry units (beams, slabs etc,) laid side by side; in common with similar prestressed concrete members, such units are likely to vary in precamber with consequent difficulties in the application of finishes. No such ‘factory’ prestressed masonry units are as yet available in this country, however. ‘Compliance with the requirements of Code Sections 3 and 4 should, in the majority of cases, satisfy the limiting deflections given. In unusual cases, to achieve these limiting deflections, itis, suggested that the designer considers reducing the stresses in the reinforcement to control both deflection and cracking. This artifice has commonly been used in the design of retaining walls by reducing the design strength of high tensile reinforcement to that of mild steel. Appendix C to the Code gives some guidance in general terms on the estimation of deflection, suggesting that an elastic analysis should be used. No guidance on the limiting values of crack width for reinforced masonry is given in the Code. However, itis pointed out that fine cracking or opening up of joints may occur in reinforced ‘masonry structures but that the cracking must not be such as to adversely affect the appearance or durability of the structure. Attention is also drawn to the need for movement joints or other precautions to deal with the effects of temperature, creep, shrinkage and moisture movement. 6 LL le wae eer sranswpioi00m gc Moverentjointin post-tensioned brickwork spareover 1000 ye atm ao ‘spans up to 10.0m be high in comparison with reinforced concrete. Cracks will tend to open only at joints between units, concentrating their size. Limiting the design strength of the reinforcement ean reduce the incidence of cracking. CLAUSE 17, STABILITY Clause 17 deals with stability. General recommendations are given regarding the need for one designer to be responsible for the overall stability of a structure and for ensuring that stability requirements are incorporated in the design of the various elements of the structure even when they may be carried out by different parties (trussed rafter roofs, reinforced concrete plate floorsetc,). Reference is also made to the need for a ‘robust and stable design’ which is particularly important in reinforced masonry structures because of the more open layouts sndbook to BSS628 2: SECTION 2: Reinforeed Masonry 0.015 Cm $5. | cose 4c, 7 7 7 Figure 2 Overall stability —_minimum horizontal loading possible in comparison with traditional unreinforced loadbearing masonry. In order to assist the designer three recommendations are given as follows : 1) Buildings should be capable of resisting, at any level, 1.5% of the total characteristic dead load above that level, acting as a horizontally distributed load as illustrated in Figure 2 2) Robust connections should be provided between elements of the structure, particularly at floors and roofs. Reference is made to Appendix C of BS5628: Part | which gives information on the use of metal anchors and joist hangers capable of resisting lateral movements, and for the forming of these connections in buildings of up to 6 storeys in height. Whilst these details can often be used without further calculations, the designer must be satisfied that the ‘connection meets the requirements of his design assumptions. In general itis also important that undue reliance is not placed upon the structural performance of individual connections because of the often variable nature of their installation. 3) Consideration should be given tothe effects ofthe differential behaviour of structural ‘lements of different materials on the connections between them. This recommendation draws attention to the variations in behaviour between masonry, reinforced concrete and steelwork etc under the influence of thermal and moisture changes. Whilst the introduction of feinforcement and grouted voids into masonry sections modifies to some degree the behaviour ‘of the masonry, the tendency of clay brickwork to long term expansion, as opposed to the long term contraction of reinforced concrete for example, should be borne in mind when connecting. different materials together, see Figure 3 may. expand reinforced masonry the use of pinned joints may avoid stress build up and cracking, but effect on stability must be considered possible racking shortens wath time reinforced concrete Figure 3 Cracking due o differential movement ‘The attention of the designer is also drawn to two constructional matters, namely the raking out of bed joints and the inclusion of dpcs, both of which may adversely affect the assumptions ‘made in the design, see Figures 4 and5 Clause 17 also refers to the stability of earth retaining and foundation structures, Although the geotechnical procedures forthe calculation of the forces acting upon these structures are not given in the Code (being obtainable in numerous suitable text books*), guidance is given on the y_ values to be used when designing reinforced masonry elements, Stability of these The brickwork in the curved wailing over the glazed entrance to this offices and shops building in King Street, Hammersmith is reinforced. \ S— sate ons width of masonry for design purposes Figure 4 Design width of masonry with raked joints TRY freestanding ‘grouted cavity wall 2 courses of ‘damp proof course bricks a8 dpc, if required, and as ‘appropriate to walling it membrane type dpc required then reduced bending and shear capaciy will result Figure 5 Damp proof course in reinforced masonry structures or structural elements will usually be governed by geotechnical criteria. The y, values to be used are the equivalent of those applied to other forms of loading, ie 1.4 for loads which. are considered to be dead loads and 1.6 for loads considered to be imposed. For the reasons discussed in Section | of this Handbook no distinction is made between adverse and beneficial loads in the application of ¥ to earth and water pressures. This considerably simplifies design procedures. There is some dissension as to the value of y, to be assigned to earth and water loads but the Code adopts a value of 1.4 when combined with dead and imposed loads, and of 1.2when wind load isalso included. As with other recent structural Codes of Practice”*, BSS628: Part 2, in clause 17 requires the designer of reinforced masonry structures to consider the effect of misuse or accident upon his design. The principle to be followed i that the extent of any damage should not be disproportionate to the cause. There is also the need to allow for particular types of damage to which structures may be susceptible by virtue of their use. Inaddition to consideration of these general recommendations, for buildings of five storeys and above design should follow the requirements of clause 37 of BS5628: Part]. The use of mortar designation (ii) in this connection is again excluded for reinforced masonry except where it onlycontains bed joint reinforcement and is designed in accordance with Appendix A of the Code. Whilst there is no theoretical reason why buildings ofS storeys and above should not have a structure composed entirely of reinforced masonry, this is unusual in the United Kingdom and elements of reinforced masonry are more likely to be combined with elements of reinforced concrete or, indeed, steelwork. The designer has to consider what accidental damage design requirements are appropriate to his structure in these circumstances. In general, the recommendations given in BS8110 and BSS628: Part | are similar. In both codes the designer is permitted to analyse the behaviour of his structure or follow the tying rules laid down. When dealing with reinforced masonry, compliance with Option 3 in BS5628: Part 1 would also satisfy BS8110 requirements for any reinforced concrete elements, since reinforcement suitable to function as ties is likely to be available in the masonry. [Rentoro BK Pant SECTION Rooney 3 2.2.3 2.2.4 2.2.4.1 ‘The final point made in Clause 17 requires the designer to consider whether any special precautions or temporary propping are necessary for the stability of the structure, or its elements, during construction. As masonry will often be reinforced to permit increases in slenderness over unreinforced masonry (retaining walls, shed type structures ete,), stability ‘considerations during construction may be more onerous than with unreinforced masonry and in such circumstances the contractor should be advised that special precautions are necessary. CLAUSE 18, LOADS Clause 18 gives definitions of the various loads to be used in the design of reinforced masonry structures, as follows: (a) Characteristic dead load, Gy for: Pobatd taken as equal to the dead load as defined in and calculated in accordance with wen 186399; Part 1, fixtures partitions (permanent) services weights to be based on unit weights given in BS648 or from actual known (including (eights tanks and contents) (b) Characteristic imposed load, Qy Load assumed to be produced by occupancy or use including: movable partitions distributed loads Concentrated loads ( ‘#Ken as equal to the imposed load as defined in and calculated in accordance with BS6399: Parts 1 and 3 or other appropriate Code of fee ioe Practice eg BS6180 for protective barriers in and about buildings snow loads (©) Characteristic wind load, Wy Load due to the effect of wind pressure or suction taken as equal to the wind load as defined in and calculated in accordance with CP3: Ch. V: Part? (4) Nominal earth load, E,, Load to be obtained in accordance with current practice, eg as described in CP2004. CP2004 in turn refers also to Civil Engineering Code of Practice No.2" for the derivation of lateral earth. Pressures. CLAUSE 19, STRUCTURAL PROPERTIES AND ANALYSIS Clause 19 deals with the structural properties of materials used in reinforced masonry and the analysis of reinforced masonry structures. Characteristic compressive strength of masonry, fy Table 3 of the Code gives values of f, for various combinations of masonry unit and mortar designation loaded in the normal manner. The values are also given graphically in Figure 1 of the Code. The information given in Table 3 and Figure 1 are included here in Figures 6to9. For those familiar with masonry design in accordance with BS5628: Part 1, obtaining f, will present few problems, indeed the process is simplified when concrete units having height to ‘width ratio of 1.0 are used, as explained in Section 1 of this Handbook, since a separate table for this case is included in Part 2. Complications may arise with the partial reinforcement of hollow concrete blockwork. However, as the sections considered to be reinforced will have their voids filled with concrete, those tables relating to solid blocks can be used for obtaining f,, provided. that the strength of the concrete In reinforced masonry itis preferable, where possible, to arrange for the units to be orientated such that the compressive force on the unit is normal to its bed face ie in the usual unreinforced ‘masonry manner. If the units are loaded compressively on other than their normal bed faces there can be a considerable reduction in the characteristic compressive strength of the units, eg if they are hollow or perforated. Where the compressive force is parallel to the bed face of the unit ic the direction of loading is ‘normal to the stretcher or perpend face, the appropriate section of Table 3can be used to obtain, the characteristic compressive strength for frogged brick masonry (filled frogs), solid units and fill is at least equal to the strength of the block. 26 24 22) $ 20 ie compressive strength of masonry (N/mm?) T Tortar desi ignation (i) t L 070 2 30 40 L 3060 70 ‘compressive strength of unit (Nm?) 30 20 700 ‘masonry constructed with bricks or other units having a ratio of height to least horizontal dimension of 0.6 dimension of 0.6. Figure 6 Characteristic compressive strength, fy, for masonry of units with ratio of height to least horizontal 26 24 22 20 13] 16 4 12 10 characteristic compressive\strangth of masonry (Nimm2) 0710 2 a0 40 dimension of 1.0 30 60 ‘compressive strength of unit (Nimm2) zo block masonry constructed with solid blocks having a ratio of hight to least horizontal dimension of 1.0. filled hollow blockwork. For cellular or perforated brick masonry the characteristic compressive strength may be obtained using the test described in Appendix D to the Code. This Figure 7 Characteristic compressive strength, fy, for masonry of solid blocks with ratio of height to least horizontal Handbook to BSS628: Part 2: SECTION 2: Reinforced Masonry m1 characteristic compressive strength of masonry (Nimm2) 38}———p - 26) ——_---___-... 34|— 32| 20] 2a| = 22 7 20 ‘mortar designation (i}| ——— 18} ——— f 7 16} 2 4 : 102004960 80 75 Bo compressive strength of unit (mm?) block masonry constructed with sold blocks having a ratio of height to least horizontal dimension of between 2.0 and 4.0 Figure 8 Characteristic compressive strength, fy, for masonry of solid blocks with ratio of height to least horizontal dimension of between 2.0 and 4.0. 2.2.4.2 test may of course be used to determine f, for any masonry materials stressed in any direction. Alternatively, ifno test data are available and the extent of the works does not warrant testing, the value obtained from Table 3 may be divided by three to give a value for f,. In the unusual case where hollow or cellular blockwork is subjected to compressive forces parallel to the bed face of the unit, the value for f, is obtained from Table 3, using the compressive strength of the unit determined in the direction parallel to the bed face of the unit in which the block will be loaded in service. Unless this information is available from the manufacturer, tests will be required to obtain the unit strength. The Code does not specify a method of determining the unit compressive strength, but the method described in Appendix B of BS6073: Part 1: 1981 may be used for this purpose 1B$5628: Part 2 also refers to units of unusual format and unusual bonding patterns and suggests that f, may be obtained for brick masonry using the method given in Appendix D of the Code, and for block masonry using the value given in Code Table 3 for the appropriate block strength. For brick masonry the value of f, must not exceed that which would be obtained using Table 3. ‘The appropriate block strength may be obtained as above using the method given in Appendix B of BS6073: Part 1. Characteristic compressive strength of masonry in bending In 19.1.2 the Code refers to the characteristic compressive strength of masonry in bending. Thisis 2 ie compressivelstrength of masonry (Nimm?) 20-— 18 mortar designation () 16}— : 14] . - 12 10 ea | ‘mortar designation (i | L 10 commmeaso 50 60 700 ‘compressive strength of unit (Nim?) block masonry constructed with structural units other than sold concrete blocks having a rato of height to least horizontal dimension of between 2.0 and 40 Figure 9 Characteristic compressive strength, f,, for masonry of blocks other than solid with ratio of height to least. horizontal dimension of between 2.0 and 4.0, 2.2.4.3 given as fj, the characteristic compressive strength of masonry, and is determined as described above. Thusit is unnecessary to factor f, when using flexural design formulae. Characteristic shear strength of masonry, f, ‘The characteristic shear strength of reinforced masonry is dependent upon whether the reinforcement in the section is embedded in mortar or concrete. The basic characteristic shear strength for both infill mortar and concrete is taken as 0.35 N/mm: ie the same as for unreinforced masonry. In both cases the value of f, may be further governed in beams by the area of the main reinforcement and the shear span (the ratio of maximum design bending ‘moment to maximum design shear force) of the beam. In the case of point loads, the shear span ‘may be taken as the distance from the support to the line of action of the point load, and for uniformly distributed loads as the span divided by 4. For combinations of loads the shear span must be calculated accordingly. In clause 22.5.2 the designer's attention is drawn to the detailing requirements when a principal load is within 2d of the support. These requirements are discussed in 2.3.6in this Handbook. 8) Reinforcement embedded in mortar ‘The characteristic shear strength for reinforcement embedded in mortar is 0.35 N/mm? in all ceases except for simply supported beams or cantilevers where the ratio of the shear span, a, 10 effective depth dis less than 2. In this case, f, may be increased by the factor “4 where avis the distance from the face of the support to the face of a principal load, see Figure 10, however, £, must not exceed 0,7Nimm?, No enhancement is permitted for uniformly distributed loads. see fig 30 all main tension reinforcement to be fully anchored at support { |W > 0.7R (pxincipal load) : Figure 10 Definition of principal load Handbook o BSS628: Part 2: SEK |ON 2: Reinforced Masonry 3 2.2.4.4 ‘The Code points out that where a wall is subjected to considerable vertical loading the wall may have adequate shear resistance when considered as unreinforced in accordance with BSS628: Part 1. In this case, the shear strength would be taken as 0.35 N/mm: plus 0.6 times the design vertical load per unit area due to vertical dead and imposed loads for the appropriate load case, upto a maximum of 1.75 Nimm?. It should be noted that the BSS628: Part I material partial safety factor Yn, of 2.5 must be applied as opposed to the value in BS5628: Part 2 of 2.0 Interestingly, the same value off, is used for racking shear in reinforced shear walls, where the basic value of 0.35 is enhanced by 0.6 times the design load per unit area (gq) for the appropriate loading condition from clause 20, ie. dead and wind load or dead, imposed and ‘wind load. However, in this case a Yq, value of 2.0 used. Again the upper limit for f, is L7SN/mm?, b) Reinforcement embedded in concrete ‘When the reinforcement is surrounded by infill concrete the characteristic shear strength is taken as 0.35 + 17.5 pup toamaximum of 0.7 Nimm? where pis the cross-sectional area of main. reinforcement divided by the effective depth times the section width, b. This shear strength can be further increased in simply supported beams and cantilever retaining walls, where the ratio of shear span to effective depth is less than 6, In this case f, is increased by the factor (2.5 ~ 0.25(a/d)) up toa maximum of 1.75 Nimm?, Where the main reinforcement in a shear wall is surrounded by infill concrete the value of f, for racking shear may be taken a8 either 0.35 + 0.6 ga, as above, or as 0.7 Nimm? provided that the ratio ofthe height of the wall to its length does not exceed 1.5, whichever method gives the greater value. Itiscommon practice to build unreinforced masonry walls off dpc on conerete ground slabs. Whilst such a practice may not be common in reinforced masonry construction, ifitisdone, the presence of a dpc will probably reduce considerably the shear resistance of the wall and should therefore be allowed for in design Characteristic strength of reinforcing stecl, fy ‘The characteristic tensile strength of reinforcement as given in the Code is reproduced in Table 1 below. Clause 19.1.4 states that the characteristic compressive strength of reinforcement should be obtained by multiplying the values off, in the Table by a factor of 0.83. In Table 4 of the Code the values off, for plain steel bars and for high yield steel bars have been transposed A future amendment will deal with this error. ‘TABLE 1: CHARACTERISTIC TENSILE STRENGTH OF REINFORCING ST (FROM BS5628: PART 2) Designation Nominal Size Characteristic tensile strength, Nim? Hot roled plain Al 250" Steel bars complying wvth BSe4a9 Hot rolled deformed all 60" high yieldstet barscomplying with BSt449 Cold worked stee! all 60 bars complying with B56! Harddrawn tee! Uptoand 485 Including 12 fabriccomplying with BS 4983, Stainless Ste! all 460 complying with BBS970: Part arades 304815, 16831 or 316833 * Corrected from Table 4in BS5628 : Part? 4 2.2.4.5 Characteristic anchorage bond strength, f, ‘The value of f, depends upon whether the reinforcement is: 4) of circular cross section with a plain surface, b) of type 1 or 2having a deformed or ribbed surface, ©) embedded in mortar, 4) embedded in infil concrete The values of f, are given in Table 2. ‘TABLE 2: CHARACTERISTIC ANCHORAGE BOND STRENGTH, f,, Reinforcement MortariConerete 50mm) ‘embedded in mortar 1s Plainbars ‘embeddedininfileoncrete 1.8 ‘embedidedin mortar 20 Deformed Bars types 1&2 ‘embeddedin infilconcrete 25 ‘The recommendations given in Clause 19.1.6 may not apply where bed joint reinforcement incorporated purely to increase the lateral load resistance of the wall. In this case most proprietary reinforcement systems rely upon the mechanical anchorage afforded by thei welded latticed form. Because of the limited thickness available in bed joints the usual means of lapping the bars will be impossible with some of these laticed systems. The ends of each strip of reinforcement should be cut to provide the longitudinal bars with as much side by side lap as possible without removing internal reinforcement or overlapping the strips of reinforcement in the vertical plane. An absolute minimum of 150 mm lap between the longitudinal bars should be provided, and the positions ofthe laps should be staggered in adjacent bed joints. The recommendations given in Clause 19.1.6 may not apply where bed joint reinforcement is incorporated purely to increase the lateral load resistance of the wall. In this case most proprietary reinforcement systems rely upon the mechanical anchorage afforded by their welded latticed form. Because of the limited thickness available in bed joints the usual means of lapping the bars will be impossible with some of these latticed systems. The ends of each strip of reinforcement should be cut to provide the longitudinal bars with as much side by side lap as possible without removing internal reinforcement or overlapping the strips of reinforcement in the vertical plane. An absolute minimum of 150 mm lap between the longitudinal bars should be provided, and the positions of the laps should be staggered in adjacent bed joints. 2.2.4.6 Elastic moduli, Em, E., Ey Beinga composite and anisotropic material, masonry, and often more so reinforced masonry, will have a value of elastic modulus, E,,, which depends upon the masonry materials used, and the direction of loading etc. For accurate determination of deflection, rotation etc. it will, therefore, be necessary to carry out tests to arrive ata value for the elastic modulus. However, in most cases it will be sufficient to use a value for Ey, of 0.9f, as given in Code Clause 19.1.7 even though the range is substantial in practice, 0.5 fy ~2.0 f, (see Handbook Section 1). Also given in this clause are elastic moduli for concrete infill, E., and for prestressing tendons, E,, for use in the calculation of losses of prestress. A further value of E,, the elastic modulus for all steel reinforcement, is given as 200 kN/mm?. This value is accurate enough for all design calculations. Reinforced brick stair tread under construction. landbook to BSS628: Part2: SECTION? Reinforced Masonry 1S Analysis of structure ‘The implication of the first paragraph of clause 19.2is that calculations will be required for a ‘number of limit states. In fact for reinforced masonry design, compliance with the span/depth ratios and slenderness requirements given in Section 4 of the Code will mean that in general only ultimate limit state calculations will be necessary. Since reinforced masonry will often be used in conjunction with reinforced concrete, property differences of the materials should be allowed for in analysis of the structure, similarly if lay brickwork and concrete blockwork are used in conjunction. The Code also requires that the analysis ofthe structure should recognise indeterminacy where appropriate and make due allowance for continuity and joint stiffness in the derivation of forces in members. Three assumptions for the calculation of the stiffness of ‘members are given in terms of the member cross-section and are illustrated in Figure 11. For ‘obvious reasons the first of these will be appropriate for the design situation. The latter two methods will be more useful when analysing existing structures. It should be noted that the three methods can give considerably different answers in certain circumstances, An o ‘Am = total masonry section ignoring reinforcement ‘Ame = compression area of masonry ‘ea of reinforcement = rato of Esto Em, aS raae ] ! \ AnBe+ (mx As) Ame +(m xAs) “i ay Figure I Alternative cross sections for derivation of relative stiffness 2.25 16 CLAUSE 20, PARTIAL SAFETY FACTORS Clause 20 deals with partial safety factors for materials (mm €te,) and for loads (,) ‘The values of used in BS5628: Part 2 are based upon those in Part 1 and the same four load cases are adopted. The Code accepts that in certain circumstances other values of y;may be appropriate eg for farm buildings where type of structure and design life are factors. Italso accepts that alternative values of yfor E may be more appropriate in the derivation of, design loads for earth and water pressure, but the designer must carefully consider the implications of amending the values given. Each of the four load cases referred to in 20.2 must be considered by the designer and the critical case or cases must be used in the design. Generally, consideration having been given to Clause 17.3 ‘accidental forces’, no further detailed design requirements involving loading case 20.2.1(d) will be necessary unless the building exceeds four storeys in height, ie itis a category 2 structure as defined in BS5628: Part 1. Itshould also be noted that when dealing with earth or water loads the Code considers it unnecessary to make any distinction between adverse and beneficial loading conditions. Thus, there is no need to consider an alternative value of yf 0.9 or 1.0 in cases (a), (b) and (c) in relation to these loads although this might appear to result in more critical loading, Because of the need for closer control and efficient design of reinforced masonry, those partial safety factors on materials which are also used in BSS628: Part I have been modified in Part 2 ‘Table 3 below gives the values of the material partial safety factors used in BS5628: Part 2, together with those used in Part | for comparison purposes. The Table gives the values Of Ym for usc in the ultimate limit state, the case of analysis for accidental damage and also the serviceability limit state, Clause 20.3.1 in the Code gives values for the partial safety factors for loads, 7, for the serviceability limit state. These were as used by CP110. No serviceability values of yor earth and water loads are given; thus some judgement is required if calculating deffection etc, involving earth or water pressures. ‘The Code rightly reminds the designer that it may be necessary, in certain cases, when dealing with the limit states of deflection and cracking, to allow for additional deflections resulting from moisture expansion in clay brick reinforced masonry, drying shrinkage in reinforced concrete blockwork and creep movements in both. All of these movements are time dependent, being at ‘TABLE 3: PARTIAL SAFETY FACTORS FOR MATERIALS “Matera patil safety factors Limit BSSe2s: ——BSSOZS: state Part Compressive strength of masonry ULS. 23,20 ta Sus. is = AD. TIS O SSSA Strength ofstee! ULS 1s sus. | AD. 10) Shear strength of masonry ULs. 20 25 I AD. 10 135 Bondstrength| ULS. 18 i AD. 10 ULS— Ultimate mit state SLS—Serviceabilcy limit state ‘AD~ Misuse and accidental damage their greatest immediately after construction and becoming of less significance, although never quite ceasing, with the passage of time. It may be noted that with BS5628: Part 1 and CP110 the selection of partial safety factors was governed, inter alia, by the desire to achieve comparable global factors of safety with those inherent in permissible stress design. Thus in CP110 global factors of safety on strength ranged from 1.61 to 1.84, and in BSS628: Part 1 from 3.5 to 5.6. In BS5628: Part 2there is no predecessor and the global factors of safety are pitched between the two ranges mentioned above ie 2.8t0 3.68. Inthe analysis of continuous members, the Code requires consideration of only two loading, arrangements as follows (see Figure 12) 1) maximum design load on alternate spans and minimum design dead load on the remaining spans, 2) maximum design load on al spans. Thus itis not necessary to consider the case with maximum design load on adjacent spans and ‘minimum design load on the remainder. be Mosc, “+ tt tot Figure 12 Load cases 23 Itshould be noted from Figure 12 that, where spans differ, case (1) may need to be carried out, twice to obtain the maximum design bending moments and shear forces. CODE SECTION 4. CLAUSE 21, GENERAL ‘The designer is permitted to assume that in the majority of cases, when designing reinforced masonry, the ultimate limit state will be critical. Thus compliance with the design guidance and particular recommendations given in the Code obviate the need for special calculations for the. serviceability limit states of deflection and cracking. These calculations, of course, can be Handbook to BSS628: art 2: SECTION 2: Reinforced Masons 7 2.3.2 2.3.2.1 carried out where particular circumstances warrant, CLAUSE 22, REINFORCED MASONRY SUBJECTED TO BENDING Clause 22 deals with reinforced masonry elements subjected to bending and covers the design of beams, slabs, retaining walls, buttresses and piersetc. It must be accepted that the design of slabs, and perhaps beams, in reinforced masonry is not very common. Itis also important to realise that the sizing of reinforced masonry elements will be controlled by the size of the ‘masonry unit and the direction in which the units are loaded, ‘Wall panels resisting lateral loads may als6 be designed in accordance with this section but, where only wind loading is involved the use of the methods in Appendix A will be more appropriate. Effective span ‘The effective span of elements follows reinforced concrete practice as shown in Figure 13. + } t ¥ I wae] | i smaler a | led fal le stance between sfecive span esol supports 2) continuous beam vse ne smaler of eftecive span {q_sflectve span al ) cantiever beam Figure 13 Determination of effective span Limiting dimensions In Clause 22.3, limiting dimensions are given to control deflection and cracking; these are assumed to result in compliance with the recommendations of Clause 16.2.2 discussed in 2.2.1 previously. It can be seen from Code Table 8 that where a wall is subjected to lateral wind loading it may often be beneficial to design to Appendix A methods since these permit greater panel sizes, except when the 30% increase permitted by this clause for free-standing walls is used, as shown in Table 4 below. However, methods one and two in Appendix A refer back to Clause 22.4.3 and itis not made clear which of the limiting dimensions requirements apply. Itis reasonable to assume that since Appendix A relates specifically to bed joint reinforced panels, the limiting dimensions in the Appendix override those given in Table &so that Table 8 may be taken as referring to laterally Joaded walls which are reinforced other than with bed joint reinforcement. Limiting span to effective depth ratios for beams are the same as those used in BS8110, as are the limiting distances between lateral restraints; this isas illustrated in Figure 14. is not defined for cantilevers but where the width of the compression face varies it may be taken as the average width. Following reinforced concrete practice, it may be assumed in the ‘case of upstand beams that lateral restraint is provided by a slab attached to the tension zone of the beam, provided that the slab thickness is at least one tenth of the effective depth of the beam and that the beam does not project above the slab by more than ten times its width. ‘The limiting span/effective depth ratios given in Code Table 9 are set to limit the deflections of beams to those values given in 16,2.2.1. Ifthe designer wishes to use a different limiting deflection of say’ 7 of a beam of span s, it is necessary only to multiply the values in Table 9 by the 18 ‘TABLE 4: LATERALLY LOADED WALLS: LIMITING DIMENSIONS. Code Tabet ‘ode Appendix A Lng densi “imting dimensions ‘mack smaximom | maximum Span oan | aren one simply | 380 0 way supported | 45.58) : span continuous | 456 on (Sis) SSH F | 2ormore W008 ioe a supportonsedges | otter OW 1001 otal emer | 2uayspan sa (ss8) tnaximum ae dormore | Ata none | "ans sides 682285 continuous supportonsedges | othercases | 60tr on? oat | 7 cantilever isa fr egons @sae) - : 2 | 85 0.00 | + FU Sinn + a,)| ‘ terete tatty Fm | | eS maximum 100mm 1 ftel ts Shon wa mew 4 ; Ih : ov ‘ertanced i he slab 7, | : simple Aver Sole = t aL + vey aKs y Tia RG Fars SECTION: Rey ‘TABLE 6: EFFECTIVE THICKNESS ty. ‘sngle eat wal cauly waltone lea entorcec routed eaviy wall ters ooatr af tee tieteste Sash tor ree bm 8) rete ot ices oat tet a qe =j Is { TL Th | 4 Trak] de | t4| SSIS | ic 29, queta bond wal tor practea reasons he renorces ite> 100mm reniorced hotow bck wallet teal usualy be he ticker lal Leteteto ‘The effective thickness of reinforced masonry walls is illustrated in Table 6. Design guidance is divided into that for columns and that for walls. Design for columns is further subdivided into short columns, short columns subjected to biaxial bending and slender columns. Walls are subdivided into short and slender walls. Short columns may be designed from fist principles, assessing the moment and axial load capacity of the column using the assumptions given earlier in the Code for bending members, ot using the design equations given. The first equation is used where the loud is predominantly axial and does not exceed the capacity of the masonry alone. A check using this equation will establish whether the reinforcement capacity must be used, in which case further equations are given for the vertical load capacity and the moment capacity of the section. In each case a rectangular stress block as illustrated in Figures 19 and 20is used. “fe = . | i 2 winere bis the width of the section Figure 19. Stress block - short column Post-tensioned reinforced brick pier under ‘minimum reinforcement case (load capacity construction. ifmasonry alone adequate) ‘The solution of the latter equations requires an assumption of the depth of the compressive stress block d,, so that the steel stresses can be obtained. This can be done on the basis of the stress strain curves for reinforcement given earlier in the Code or using the guidance given in clause 23.3.1.1 and illustrated in Figure 21. Design is thus an iterative process involving assumptions of d, and estimation of stee! stresses which give load and moment capacities which exceed the design values. Design charts based on these equations and the stress strain curves given in the Code, are included in section 4.0 enabling the design process to be simplified. An alternative method of design when bending predominates, permits the axial load to be ignored and the section designed as a beam resisting an increased bending moment. The resulting area of steel may then be reduced to allow for the vertical load effect. ‘The Code states that in general itis only necessary to consider single axis bending, even when ‘moments occur about both axes. Asy Age (area of stoe!) ts 32 ‘column(least Vina Vs (tee! sxese) ‘comoressed face) competi seed ss a sxinimum A permits ‘sho cece |e foaseu 4 te : (dp eee>! tension a r i {> tte) es tar ghee Fst) 922 ay 0st of Aalto | eonsng — 1 Figure 0_SivssBlack—short column design Figure 21 Variation of depth of compression Block, de reinforcement case wh sressinreinforcomentin least compressed face of column, fia However, in certain circumstances, eg square corner columns with substantial moments about both axes, it may be necessary to design the column for biaxial bending, BS5628: Part 2 enables thisto be done by increasing the design moment about one axis, the axis being that which has the greater ratio of the design moment about the axis to the column's overall dimension perpendicular to the axis. Thus the design moment about the critical axisis increased by the ‘moment about the other axis factored by the ratio of the column dimensions and a factor which depends upon the ratio of the axial design load to axial design capacity. There are two main errors in this clause which will be dealt with in the first revision to it. Firstly the ratio of the column dimensions in the expression for the increased moment about the y axis must be inverted, and secondly the expression for the design axial load resistance of the column, Nap, should not include any contribution from the reinforcement or the partial safety factor for compressive strength of masonry. The correct expression is Nae=fc Am where Am isthe plan area of the column. ‘The whole section dealing with column design will be amended in the first revision to the Code to resolve the present anomalous use of the word ‘axial’. The use of the word ‘vertical’ might be ‘more appropriate in place of ‘axial’ in some instances, since eccentricity is also covered, see Appendix A. ‘Sub-clause 23.3. 1.3 gives a method for the design of slender columns, ie columns with a ratio of effective height to column dimension inthe plane of bending in excess of 12. Design must allow for the additional moment which s deemed tobe induced by lateral deflection being function ofthe design ond, Nth etfective height hy and slendernes het as with short columns, design may then be caried out from first principles, using the basic assumption given inthe lause on analysis of section 22.4.1), or using the equations for short column design (the design moment including the additional slenderness moment) or using the design charts. In each case the design moment includes the additional slenderness moment, My, of the columns, given by = Nhe? Ma ae ‘The apparent discontinuity at a slenderness ratio of 12is compensated for by the minimum. design moment of My = N x 0.05. Guidance on the design of walls resisting combined vertical loading and bendingiis limited in the Handbook to BSS628: Part2: SECTION2: Reinforced Masonry] 28 2.3.4 23.5 23.6 Code to the use of the basic assumptions given in 22.4.1 for determination of vertical load and bending resistance. As for columns, a slenderness ratio of 12 defines the change from short to slender walls, the latter being designed for an additional moment obtained as above forslender columns, Because reinforced masonry walls will usually be singly reinforced the column design equations are not referred to, but there is no reason why they should not be used where appropriate, or if suitably modified. Where the resultant eccentricity exceeds 0.0, the column equation cannot ‘be used and the Code recommends that the walls are designed as flexural members, neglecting the axial load, Whilst compliance with the Code recommendations generally satisfies serviceability requirements, Code clause 23.5 draws attention to the case of lightly loaded columns subject to bending ie where the design stress due to vertical load is less than f,/2, considered as a uniform compressive stress across the section. In order to control flexural cracking the Code recommends that the column is treated as a beam and reinforced in accordance with the detailing section of the Code. CLAUSE 24, REINFORCED MASONRY SUBJECTED TO AXIAL COMPRESSIVE, LOADING This clause deals with axially loaded walls or columns, ie walls or columns in which the eccentricity of load does not exceed 0.05 times the thickness of the section in the direction of the eccentricity ‘The clause recommends such members are designed in accordance with BSS628: Part 1, ignoring the reinforcement, or with clause 23.3. 1.1, using the equation for vertical load resistance, Ng, given in part (b) of that clause. Thus there is no automatic allowance for a minimum eccentricity in the design of these members. CLAUSE 25, REINFORCED MASONRY SUBJECTED TO HORIZONTAL FORCES INTHE PLANE OF THE ELEMENT Clause 25 deals with reinforced masonry shear walls. Bendingin the plane of the wall is unlikely tobe critical, although the compression in the end of the wall due to bending should be checked. In walls carrying low vertical loads tension may occur at one end and reinforcement should be provided accordingly. ‘The Code concentrates on the racking shear design of shear walls permitting the assumption that the horizontal shear force is uniformly distributed across the plan area of the wall. The design shear strength of the masonry is obtained from clause 19.1.3.2, as discussed in 2.2.4.3 of this handbook. ‘Where the design shear strength is exceeded, horizontal shear reinforcement must be provided such that the area of shear reinforcement per unit height of wall (A,.) divided by its spacing (s,) is greater than, or equal to: Ay tf /ym) Sv fyYms. where tis the thickness of the wall and v the shear stress due to design loads. v must not exceed 2.0m As discussed above tension and compression in the ends of the shear wall due to horizontal forces should be checked and,where out of plane bending occurs, it will be necessary to design the end of the wall carrying the maximum compression as a column. The Code specifically refers to the need to treat the end of the wall as a slender column where the slenderness of the wall at right angles to its plane exceeds 12. CLAUSE 26, DETAILING OF REINFORCED MASONRY ‘The sucessful use of reinforced masonry and its future prospect as a major structural material are largely dependent upon the development of a simple and efficient system of detailing. Code Clause 26 gives the basic principles on which such a system must be based. The variety of masonry units and forms of construction available make generalisation difficult. Thus no explicit minimum area of main reinforcement is given in the Code, but the designer is advised to consider whether design to Part I of the Code would be more appropriate when the area of reinforcement isa small proportion of the gross area of the section. This, of course, relates to walls and columns since beams etc., are not covered by BS5628: Part 1. Conversely, as pointed out previously, where the masonry simply provides formwork to the concrete, design to BS8110 may be more appropriate. ‘The maximum size of reinforcing barsis limited, for practical reasons, as shown in Figure 22. | ramet at Qh neces bed joint reinforcement SEAS ©) quetta bond Pocket-type reinforced brick retaining wall - {forming bridge abutment and parapet. Q ‘ oo maximum bar dameter SIN motto erceed Samm masonry beam WSN 4) coumn bar diameter in cases a) t 1 notto exec 25mm an Figure 23 Pocket wall maximum reinforcement Figure 22 Maximum reinforcement sizes size Pocker-type reinforced brick retaining wall under construction at Banbury ~see inside front cover. ndbook to BSS628: Part 2 IN2: Reinforced Mason 27 = ‘sp maximum aggregate size +5mm |° 8 9? 500mm A oc beraaneet or Onm wnchever i greater ° o ® 6 fe 2 or ife>2e, 12 x bar diameter ord whichever is greater SS | bonding moment envelope WY \ i ‘curtailed bar no longer required ) to let of ine moment capacity of reinforcement shear force diagram—~ Figure 29 Reinforcement curtailment Ata simply supported end each tension bar should be anchored as shown in Figure 30, a) orb), ‘except when there is a principal load within 2d of the face of the support in which case all the tension reinforcement should continue beyond either the centre-line of the support or aline¢ 2 from the face of the support, whichever is the shorter distance, to provide an equivalent anchorage length of twenty bar diameters, see Figure 31 0 Reinforced brick staircase following curved diaphragm wall for a brickmaker's head office. I t e € a z f (Ce ail FT ABD io ans Coe eae where anchorage lengih d= effective depth — cy ; + (Got TAR t “i Si gd d g oe tes olteg 0) wi support Figure 30 Reinforcement anchorage, simply supported ends principal oad 20 xbar dameter hod tt 4 or whichever is less Figure 31 Anchorage of reinforcement where principal loads occur. 31 3.0 DESIGN PROCEDURES 32 GOOD DESIGN Good economic design of reinforced masonry is dependent upon the achievement of a balance between the limitations of form imposed by masonry unit size, and the need to minimise section size and complexity of construction. A working knowledge of masonry bonds and nominal and ‘work sizes of units is essential in this respect. The BDA publishes a document® which lists coordinating dimensions for brick masonry units and the concrete block manufacturer's technical literature offers similar guidance. ‘The decision whether to use reinforced concrete clad in masonry or reinforced masonry inits, own right will often be governed by non-structural requirements (eg aesthetics). However, there will be many instances where the use of reinforced masonry for the major part of a structure or for individual elements is structurally and economically beneficial to all parties. ‘There is no reason why reinforced masonry should not be used in association with reinforced concrete and structural steelwork where such an arrangement is advantageous either structurally or economically. With regard to the appearance of various bonding patterns itis interesting to note thatin the mid 19th Century the outer leaf of cavity walls was built using snap headers to give the appearance of Fiemish or English bond because stretcher bond was considered dull and uninteresting. Aswith reinforced concrete, durability isa primary design consideration which must be dealt with at an early stage in the design of reinforced masonry. Although durability is not dealt with in detail in this document, aspects are discussed at the beginning of Section 5.0. DESIGN OF FLEXURAL MEMBERS ‘The structural elements most commonly designed in reinforced masonry are probably beams and walls etc, which act in bending. Although the design principles follow those of reinforced concrete, the unit size and type and bonding pattern influence how the reinforcement is incorporated into the section. A number of design procedures are possible depending upon whether the designer is considering alternative masonry types, or alternative masonry strengths. A representative procedure is illustrated in Figure 32. Above left: Pocket-type reinforced brick retaining wall and parapet for a bridge. Above right:Deep corbel in reinforced brickwork. Below: Reinforced brick arch for an office block. 32 masonry unitiyper int ‘durability? design bending eee een catulate design bering compressive calculate moment tO @ ©, yes excessive + fa calculate Aa required cata design shear HG) se ¥ = provide design shear links provide nominal Tinks. Figure 32 Typical flexural design flow chart notes bricks, blocks, hollowlsolid mortar, concrote {quetta bond, reinforced hollow blocks, grouted, cavity ete ‘based on unt size and bonding. check span/eft. depth rato and lateral stabilty i required ‘may affect of. depth, choice of infil and reinforcement type bending use charts or code equations shear if required provide links, if bonding can accommodate them if not increase section size ‘nominal links required at iserection of designer detail in accordance with ‘cod, og curtailment, anchorage cna 8S Pn SECTIONS Ro oo] 33 3.3 It should be noted that itis possible, where circumstances permit, to simplify beam design by providing a section of sufficient depth that shear reinforcement may be omitted. This has the advantage that the limitations imposed on the positioning of shear reinforcement by the bonding pattern are avoided. Whatever bonding pattern is adopted, anything other than bed joint reinforcement in brick masonry is likely to require the use of half bat bricks in order to provide suitable continuous horizontal voids into which reinforcement may be introduced. ‘Similar vertical voids may be required to accommodate links, although by careful arrangement of bonding pattern and limiting the link bar diameter, its possible to place the links in continuous vertical joints, see Figure 33. The use of special clay bricks with cut outs to accommodate reinforcement can also minimise the need for continuous voids and the cutting of bricks. Itis common practice to provide additional secondary reinforcement of small diameter in the perpend joints ofthe soffit brickwork to flexural members; this reinforcement being later bent. into, or over, the main reinforcement cage or infill concrete. Whilst this may be efficacious in providing additional support to the soffit brickwork, the secondary reinforcement must comply with the durability requirements of the Code and must be arranged in a manner that does not interfere with the placing and fixing of the main reinforcement. To avoid the risk of bi-metallic corrosion, contact between stainless stee! and mild or high-tensile steel reinforcement should be avoided. DESIGN OF COLUMNS: Column design to the Code varies depending upon whether the column is short or slender, is axially loaded, or has uniaxial or biaxical moments applied to it. Axially loaded columns will generally only occur where stiff beams, of approximately equal spans and carrying approximately equal loads, frame into the columns on opposite sides along ‘one or both axes giving a resultant eccentricity not exceeding 0,05 times the column dimension in the appropriate direction, ‘Such columns may be designed as unreinforced in accordance with BS$628: Part 1 or in accordance with the Code provision for columns subjected to a combination of vertical load and bending, the formulae being modified accordingly. With the axial load calculated, atrial section, size isselected which will be governed by masonry unit size and bonding pattern. An initial check using the design axial load capacity of the masonry section alone, without taking any slenderness reduction factor into account, will indicate whether reinforcement is required or not. If reinforcement is quired the slenderness ratio must then be calculated to see whether the column is defined as short or slender, ie whether the slenderness ratio is greater or less than 12. The effective height can either be obtained from frame analysis ofthe column and its associated beams, or more simply from Code Table 11. Where the column is short, the equation for the design axial load resistance Ng, in Code clause 23.3.1.1(b), may be simplified, the full section being in compression and the symmetrically placed reinforcement being equally stressed in compression, as follows: Ng =— or + 283 a, a Yims where A, is the total area of reinforcement ‘This equation can then either be solved for A,, having assumed a value for f,, or solved for fy, having assumed an arca of reinforcement. Its finally necessary to establish whether link reinforcement is required, based on percentage area of reinforcement, in accordance with Code clause 26.5.3. ‘Where the slenderness ratio exceeds 12, axially loaded columns must be designed for an additional moment, M,, as given in Code clause 23.3.1.3. In this case the design for axially loaded columns becomes as for short columns subjected to bending, with M, the design ‘moment. Design of short columns subjected to vertical loads and bending about one axisis an iterative process. After checking that design reinforcement is required to Code clause 23.3.1.1a), itis necessary t0 use the formulae for the design axial load resistance, Ng, and the design moment of resistance, Mg, given in Code clause 23.3.1.1b). In order to use these formulae the designer is Tequired to choose a value for the depth of masonry in compression, d.. The designer also has to make a first estimate of the reinforcement area to be provided. In choosing a value for d. the relative sizes of the design moment and design vertical load are helpful, Where the design vertical load is substantially more than the capacity of the unreinforced masonry section, d ‘would be chosen either as equal tothe full depth of the section, t, or close to it, bearing in mi Reinforced and prestressed brickwork were used extensively inthis headquarters building fora ‘major brick company. that the moment capacity ofthe column will be effectively limited to the middle term of the equation given for My, Conversely ifthe design moments large, a value of dof 5 may be chosen to gain the maximum moment resistance from the reinforcement. This will, however, limit the vertical load capacity to less than half that of the full masonry section considered as tunreinforced. By a process of adjusting d, until Nq and My exceed N and M respectively, or by increasing the assumed area of steel ifthe former cannot be achieved, a satisfactory solution can bbe obtained, Itshould be remembered that the maximum bar size permitted by the Code is 25mm and it may be necessary in some cases to try more than one section size. The process is not as protracted as it may be seem as the designer will rapidly gain a feel for the relationship between the variables. ‘The design of slender columns subject to bendingiis straightforward. The additional moment. for slenderness effects given in Code Clauses 23.3.1.3is always-calculated in relation to the ‘width of the column in the plane of bending so that if the design bending moment is about the major axesso will be the additional moment, even if the column is not slender about the major axis. Thus the additional moment is added to the design moment and design proceeds as for a short column subjected to bending. ‘The design of short columns subjected to biaxial bending to Code clause 23.3.1.2is also fairly straightforward. Having assumed material properties, section size and area of reinforcement, the design axial load resistance of the column, Nac, iscalculated and the value of the coefficient exobtained from Code Table 12. Depending upon the relative values of the ratios of the design moments about the two axes to their respective depth of section in their directions of bending, an increased design moment M,’ or M,’is calculated about the critical axis. Where, for square columns, the design vertical load, N, is low in relation to Ng,, the axial load capacity of the column, the increased moment tends to the sum of the moments about the two axes. Having “obtained the increased moment the procedure isas for uniaxial bending using the increased moment, Link reinforcement is only required in reinforced masonry columns when the total area of the main steel exceeds 0.25% of the area of masonry. In practice links will usually be required. The links will in general be accommodated inthe bed joints of the masonry and will, therefore, be limited in size to 6mm diameter. Link spacing is given as the lesser.of three criteria, namely, the minimum plan dimension of the column, fifty times the link diameter or twenty times the main bbar diameter. Assuming the use of 6mm diameter links and main bar diameter not less than 16mm, fifty times the link diameter will govern to give link spacing of 300mm in most cases for brickwork, and at 200mm or 225mm depending upon block unit height, for blockwork. vAbook to BS5628: Patt: SECTION2: Reinforeed Masonry 35 [40 DESIGN CHARTS 41 SINGLY REINFORCED BEAMS Mibd? fy, 0 200 250 = 300 350 400 450 500 y Yen Mibd? t, 500 ” 42 ‘COLUMNS Nibt fy 03 0.4 oe Mbt? f, 0.6 dit = 0.75 fy = 460N/imm? 2.0 Nibt fy 0.1 0.2 0.3 0.4 0.5 06 Mibt? fy Handbook to BSS628: Part 2: SECTION 2: Reinforced Masonry 39 Nibt f, 0.2| by TRS NEN h t 08 = 460N/mm?| aan ye 23 As 2 Fbt i —}——8.0 x 10°F t |2.0 x 10% tote 1.0x 10°) 0.3 Mibt? fy 08 Nit ty 0.6) 0.4] 0.2 dit = 0.85 fy = 460N/mm2 = 2.0 Vane 23 te 3.0x 109 25x 10% 2.0x 10° 15x 109 1.0x 109 0.5x 10 0.4 0.5 06 Handbook to BSS628; Part2 :SECTION2: Reinforced Masonry a Nibt fy 0.2 hy 0.4 Mibt? fy 12.0 x 10% 1.5-x403, 1.0x 10°) 0.6 Nibi fy 2.0 x 103 14.5-x40-3 1.0.x 10°) 0-6-4103) —0 \, Y y y 0 0.41 0.2 03 Mibt? fy, 04 05 06 [nihok ash SECTION Retr awa 5.0 REINFORCED MASONRY DESIGN EXAMPLES 5.1 SAL Note thatin the following examples the minimum grade of infill concrete (25) permitted in the Code has been used. Careful consideration should be given tothe selection of infill concrete to ensure adequate durability for the reinforcement type and exposure situation in question. The Code guidance is likely to be upgraded in a future amendment. EXAMPLE 1, SIMPLY SUPPORTED BEAM Design a simply supported reinforced brickwork beam required to span 4.5m carrying a characteristic dead load of 20.0 kN/m (including self-weight) and a characteristic imposed load 0f6.0kNim. +) inrersnain tse AY [___48 Materials Bricks—unit compressive strength = 35 Nimm? Mortar—designation (ii) Characteristic compressive strength of masonry, fy = 9.4 N/mm? Characteristic tensile strength of reinforcement, f, = 460 Nimm? Infill concrete grade (BS5328)25 N/mm? Partial safety factors Loads (y): Designload = 1.4Gk + 1.60, Materials: Masonry compression, Ymm=23 (normalcategory of manufacturingcontrol) (Note, many manufacturers now meet the requirements of the special category of manufacturing control, enabling Yn to be reduced to 2.0). Masonry shear, Yoo = 2.0 Reinforcement to infill/mortar bond, mp = 1.5 Reinforcementstrength, Y= 1.15 dead load, Gy = 20.0kN/m posed load, Qy = 6.0 kN/m Design load = 1.4 x 20.0 + 1.6 x 6.0 = 37.6kNim Design 2 ‘Simply supported design moment, M, = 28% 4:5" _ 95 7kNm Design shear fore, ¥, = SES48 Assuming that there are no non-structural beam depth requirements, the effective depth will be based upon the limiting span to depth ratios given in the Code, the coordinating dimensions of brickwork coursing and possibly the shear strength of the beam, Tn order to suit column width make beam width, b, = 440 mm. Use bricks as soldier courses for aesthetic reasons and so that brickwork in compression isloaded normal tits bed face. (Note that if brickwork is laid horizontally such that itis loaded in compression on its perpend or stretcher face, femust be determined in accordance with clause 19.1.1.4 of the Code). ‘Try section as shown. Provision of vertical voids at 300 mm centres to accept links means shear strength is unlikely to control depth. ‘Beam depth = 553 mm Assume Exposure situation E1 with grade 25 concrete cover to reinforcement = 20 mm Effective depth, d = 553~103-20-10-12.5 = 407.5 mm 540 om Spaneetve depihmustntesceed 2, ths min.efectvedepth = 820-25 mm This is less than 400 and therefore satisfactory. Design moment of resistance of beam in compression fgbd? 9.8 x 440 x 460? Ma=04 = 0.4 x 28% 0% 40" 151m 23 Yom ‘This exceeds design moment, 95.2 kNm \,\Ltinks at 200mm crs secondary 6mm ‘iameter links Design moment of resistance of beam in tension Myx Debt Ye Assume lever arm, 7, = 0.75d, equate Mato M and solve for area of tensile steel A. = 95.2% 105% = 460 x 0.75 x 400 “Therefore use 2 no. T25 diameter bars (982 mm?) 0.5 x 794 x 4602.3] _ af ~ Mx 400x941 a]=0-784 ‘Thisishigher than assumed value thus2T25 bars satisfactory, TI reinforcement percentage of Ay = 794 mm? area ofsteel gives “Handbook to BSS628: Part 2: SECTION2: Reinforced Masonry 48 982.x 100 440x400 ‘The Code does not give minimum percentages of reinforcement because iti often the case that the greater depths and widths used in reinforced masonry beams, require relatively small areas of reinforcement. Shear design = 0.56% ¥v Shearstress due to design loads, v,= 7 84.6 x 10° 2 Do d09. = 9-48Nimm Characteristic shear strength of masonry, f.,=0.35-+ 17.5p where p =A? = 0.0056 trom above) ).35 + 0.10 = 0.45 Nimm? ‘Therefore fy = Forasimply supported beam where SHES°SP80:2) istessthan6 fy may be increased by a factor [2.5-0. 25@)] M_ 95.2 1.3m 2.8; which is less than 6 Enhancement factor = 2.5 ~ 0.25 x 2.8=1.8 ‘Therefore increased f, = 1.8 x 0.45 = 0.81 Nimm? ‘Thisis less than the maximum 1.75 N/mm? andis therefore the value to use. fv Designshearstrength ofbeam = = 9.51 0.4mm? “Thisisless than the design shear stress and shear reinforcementis required. Shear reinforcement Shear reinforcement must be provided to satisfy the relationship: Cross-sectional area of reinforcement resisting shear, Aw ‘spacing of shear reinforcement ,s, > DY fat) Ys Ym ‘ys dictated by spacing of vertical voids ie 300mm centres; thisis equal to 0.75 d and therefore acceptable. 440 (0.48~0.4)1.15 300 _ 4g 2 250 ee Check nominal area of shear reinforcement required from Therefore Ay required = Aw —0.002b, By ‘Thus Aw = 300 x 0.002 x 440 = 264 mm? ‘Use R10 links in pairs at 300mm centres (Aw =314 mm?) twill be noted that additional secondary stainless steel reinforcement is shown in the section to, assist in the support of the masonry soffit. Alternative design using Code Table 10 and Figure 3 Inorder to simplify the above iterative process, the Code includes. table and chart from which the lever arm factor, c, may be obtained directly. ‘Thus writing Mg in terms of Qbd? and rearranging 5.1.2 Yom Therefore, from Code Table 10 or Figure 3, 7Bandz = 0.78 x 400 Awhyz Yon 295.210 1.15 460 x 0.78 x 400, and from My= Ay 763 mm? ‘Thus2no. T25mm diameter bars are satisfactory (A, = 982mm?) Alternative design using design charts From the singly reinforced beam design chart with “Ynu = 2.3, given in Section 4.0, the area of tension reinforcement may be obtained directly as follows: M 95.2 x 10° ee = ee = 0.104 bah, ~ 440 x 400? 9.4 fromchart fort, = 460 Nimm? p= t= 4.4 x 10-4 fy ‘Thus Acrequired = 4.6 x 10-* x 440 x 400 x 9.4=761 mm? As before 2no T25mm diameter bars are satisfactory. EXAMPLE 2, CANTILEVER BEAM Design a reinforced brickwork cantilever beam to carry a canopy structure using Code Table 10. ‘The characteristic dead load is 10 kN/m, including beam self weight, and the characteristic imposed load 3.6 kN/m, uniformly distributed, with a characteristic dead point load at the end of, the 2.4 m span of 6.0kN. Calculate the deflection at the end of the beam. ‘canopy Gc 10.0KNm Gx=6.0KN Oh 3.6KNim Materials and partial safety factors as for Example 1 Design Make width 327 mm to suit pier width ‘Make section depth 553 mm, as shown, d= $53~20-8~ 10=515 mm (cover20 mm) Effective span = 2400+515/2. = 2657.5 mmsay 2660 mm Cantilever span/effective depth ratio = 7.0 ‘Therefore, minimum effective depth d = 2660/7 = 380 mm ‘This s less than 515 and therefore satisfactory. Note, to ensure lateral stability the clear distance from the end of the cantilever to the face of the Handbook to BSS628: Part 2: SECTION 2: Reinforced Masonry 47 Support must not exceed 25b, or 100b,7!d whichever is less. 2b. =25x327 =8175 mm 100b.? _ 100 x 3277 a 315 20763 mm_ Both exceed 2400 mm, therefore, the beams laterally stable. 227 553 515 ‘Camm diameter links secondary reinforcement Design moment, Ma, 1.451004 1653.6 256 +14360%2.66=52388m Design shear force = (1.4 x 10.0+ 1.6%3.6)2.4-+ 1.4% 6.0= 55.8kN Using Code Table 10 or Figure’3 92.310 1.06 Om axa & 94 Yom = 23740 ‘Therefore c = 0,85andz = 0.85 x 515 Asf2 and solving for A, Rearranging Code equation Mz Ye 92.3. x 10° x 1.15 460 x 0.85 x 51: Use 2no. T20 diameter bars (628 mm?) A, 528 mm? Shear design v Shear stress due to design loads, v, = py 10% S15 0.33 N/mm? os, 327 x 513 £,=0.35 +17.5p =0.35 + 17.5 0.41 Nimm* No shear strength enhancement is possible because Code Clause 19.1.3.1.2 permits enhancement forsimply supported beams and cantilever retaining walls only. f& _ Od Yow 20 Thisis less than the design shear stress and shear reinforcementis, therefore, required. 21 Nimm? Design shear strength of beam =. 52 5.21 Shear reinforcement ‘Assuming voids at 150 centres (giving link spacing less than 0.75 4) Agg = 3210.38 = 0.21) 11S x 150) 28 mm? Check nominal area of shear reinforcement required ‘Age = 150 x 0,002 x 327 = 98 mm? R8diameter links are satisfactory (Aq = 100 mm?) Deflection Calculation of deflection of members in bending is not generally necessary providing the span to ‘effective depth ratios given in the Code are complied with. However, the Code suggests that as an alternative, the designer may calculate deflection and gives some guidance in Appendix C. Use elastic deflection coefficients for cantilever : For uniformly distributed load, w, deflection of cantilever wit set Forpoint oad, W, deflection of cantilever =¥ > LW, 3E QI Partial safety factors (serviceability limit state) Dead and imposed load Design load = 1.0G, + 1.00, Consider long term deflection. From Appendix C, Em = 0.45 fy = 0.45 X 9.4= 4.23kNimm? bd? _ 327 x 553° ae where d, is the overall depth of the section. Design load UD load = 1.0 x 10.0 + 1.0 3.6= 13.6kNim 0x60 Ie. .61 x 10 mm* Point load = 6.0 X 2.66? x 10! Long term deflection = 13:6 2.66" x 10 — Berm deflection = 54.23 x4.61 x 10% * 3x 4.23% 4.61 x 10" = 4.36 + 1.93=6.29 mm ‘Maximum permitted deflection = length/125 Calculated tong term deflection well within permitted deflection and, therefore, satisfactory. COLUMNS EXAMPLE3, AXIALLY LOADED SHORT COLUMNN (see Appendix A for proposed Code amendment) Design a3.0 mhigh reinforced brickwork column to carry an axial design load of 1100 kN, using 20 Nimm? bricks in mortar designation (li). ‘Try acolumn section $53 mm square 4 N/mm? for 20 Nimm? bricks in mortar designation (i) f, = 0.83, = 0.83 x 460 = 381 N/mm? (using high tensile reinforcement) Partial safety factors (materials) Masonry compression, ‘Yam = 2.3 (normal category of manufacturing control) Reinforcement strength, ms = 1.15 andbook to BS628: Part2: SECTION2: Reinforced Masonry 0 Slenderness ratio ‘Assume top and bottom of column have lateral supports restricting movementin both directions. ‘Therefore her = 3000 mm tee = actual thickness = $53 mm Slendemness ratio = 329 Thisis ess than 12 therefore the column is short. Design Toallow for reinforcement use method given in Code Clause 23. Naz ba+ fan _ fede Yom Yow Ys As there is no moment applied to the column, the reinforcement in both faces of the column will be in compression and, assuming symmetrical reinforcement, will be equally stressed, thus d= ‘Thus equation becomes fcbde | (Asi + As) N ‘Yom Yme Design load Ng=1100kN Reinforcement required Rearranging equation from Code Clause 23 with A, = Ay + Aa fe Are Nast 6 Ys Therefore A, Use 4 no. T16mm diameterbars. (A, = 804 mm?) Check iflinks are required (ie if0.25%Am 628 “20 1S 1S = (882.4 + 208.5 ~ 251.2) 10° = 339.7 x 10° N = 339,7kN OSf, 0.83 f, fe ba. (t= de) + 28344, (0.514) + Yom Ys Te FromM, Aao(0.5t~da) 0.83 x 460 LIS Ma 05222. 402204820) + 628(0.5 x 440-155) +i x 280.5 400-158) = (42.14 13.6 + 16.3) 108 Both Ngand Mgexceed N and M,’ respectively 2.0 x 10°Nmm = 72.0kNm_ and the section is therefore satisfactory 35 53 53.1 Link reinforcement 0.25 x 440" 100 A,exceeds 484 mm? and links are required 0.25% of areaof masonry 484mm? ‘Use 6 mm diameter links (maximum diameter permitted in bed joints) ‘maximum spacing = lesser of: a) leastcolumn dimension = 440mm b) 50x link diameter = 300mm ©) 20x mainbardiameter = 400mm. ‘Therefore link spacing = 300mm ‘RETAINING WALLS EXAMPLE7, GROUTED CAVITY RETAINING WALL Design a reinforced brickwork grouted cavity retaining wall support 2 3.0m height of cohesionless soil of 1600 kg/m? density and an angle of internal friction of 30°. Materials Bricks ~try clay bricks with unit compressive strength 35 Nimm?, Mortar—designation (ii) Characteristic compressive strength of masonry, fy = 9.4N/mm? Characteristic tensile strength of reinforcement, f = 460 N/mm? Infill concrete grade (BS5328) 25 Nimm? Pg» 1600kg/m3 ean o=30 Pp v Partial safety factors (materials) Masonry compression, ‘Yom = .3 (normal category of manufacturing control). Masonry shear, Yq 2.0 Reinforcement strength, Ys 115 Lateral loading FromCP2‘for = 30° 8=0 Ky =0.33 Lateral load, Ey = Ka p.gH x HI2 0.33 x 1600 9.81 , 3? 7000 Stem design Loadcase a) clause 20.2.1 Design load = ye, = 1.4 X 23.3 = 32.6kNim = 32.6kNm/m From Code Table 8 minimum effective depth 23.3kN/m Design moment, M= 32.6 x 1. a 66.7 mm 18 tryd = 265mm > 166.7 mm Fromclause 22.4.2, ever arm,2= d]1 - 2-5Asf"¥mm bd fYme Try T12 reinforcement at 250mm centres, A, = 452mm?/m |, — 0-5 452 x 460.x 2.3 1000x265 9.4 x 1.15 = 265 x 0.917 = 243mm ‘Therefore,z Actyz _ 452 x 460 x 243 Yom LIS 10 This exceeds the design moment ands therefore satisfactory From clause 22.4.2, Mg = 43.9kNmim To ensure that the section is under-reinforced check that the design moment of resistance based on brickwork compression exceeds the design moment of resistance of the reinforcement. Mig = Wet faba? _ 0.4 9.4 1000 2657 _ 114.8 kNm/m Yom 2.3% 10° ‘Thisis greater that 43.9 kNm/m ands therefore satisfactory. ‘No minimum area of main reinforcement is specified in BS5628: Part 2 but a minimum of 0.15% bd seems sensible in this situation, ‘T12barsat 250 mm centres = 452 mm?, thisis greater than 398 mm?, therefore satisfactory. Code Table 10 or Figure 3 may be used to avoid the need fora trial estimate of reinforcement. Ma= Qbd? this must be greater than M 32.6 10° Therefore Q = 226X104 46 OF F000 x 265° =24e, Yom 23741 From Code Figure 3leverarm factor, , = 0.94 ‘Therefore = 0.94 x 265 = 249 mm 2.6 1.15 x 10° mm/m 460 x 249 oe : but from above provide minimum area of reinforcement = 398 mm?/m ‘Therefore use T12.at 250mm centres, Shear clauses 19.1.3.1.2and22.5.1 Design shear force, V = 32.6kN/m 32.6x 1° 1000 x 265 From clause 19.1.3.1.2 characteristic shear strength of masonry, 452 esses [xan] ‘Therefore, shear stress due to design load: 0.12N/mm? 38 Nimm? =-M_26 Shear span of wall,a, == 32:6 37 5.3.2 and f, may be increased by a factor (2.5 ~ 0.25 X 3.77) = 1.55 therefore f, = 0.38 x 1.55 = 0.59N/mm? & Yom Design shear resistance of wal 82 ‘= 0.29 Némm?, this is greater than 0.12 Nimm?, therefore satisfactory, no shear reinforcement required. Secondary reinforcement clause 26.3 ‘Minimum area of secondary reinforcement = 05 05% bd X 1000 x 265 = 133 mm"/m 100 ‘Therefore provide T10 bars at 500 mm centres (A, = 157 mm*/m) Overall stability ‘The overall stability of a reinforced masonry retaining wall should be assessed in accordance with standard soil mechanics methods, using partial safety factors equal to 1.0 and providing. overall factors of safety against sliding and overturning. Bearing pressure should be assessed in the same manner and compared with the safe bearing capacity ofthe soil forced concrete base should be carried out in accordance with BS8110, using the pressure diagram, derived with the appropriate partial safety factors, to calculate the bending moments. Typical calculations are given in the BDA handbook “Brickwork Retaining Walls” EXAMPLES, QUETTA BOND RETAINING WALL Design a Quetta bond reinforced brickwork retaining wall, 1.5m high toresist a design overturning moment of 3.9 kNm/m and a design shear force of 7.8 kN/m. What isthe maximum. ‘height of soil a wall ofthis section may retain, assuming the soil to have the same properties as 328] rl la = 164 those given in Example 7 above? 1500] vd .0f50 N/mm? compressive strength isused ina mortar Characteristic compressive strength of masonry, fy = 12.2 N/mm? Characteristic tensile strength of reinforcement, f, = 460 Nimm? Note austenitic stainless steel reinforcement required for durability. Pockets are inflled with mortar designation (ii. Partial safety factors (materials) Masonry compression, Ymm = 2.3 normal category of manufacturing control). Masonry shear, Yny = 2.0 Reinforcement strength, Yins = AS Bondsstrength, Y= 1.5 ‘Stem design — using Code Figure 3 Design moment, M = 3.9 kNm/m M __3.9%10° Therefore; ‘bd? = 7000 x 1642 = 0.145 & 4 5.3 Nimm? Yom From Code Figure 3, c> 0.95, therefore usec = 0.95 ‘Therefore lever arm,z.= 0.95 x 164= 155 mm 3.91.15 x 10° mm/m 460 x 155 2 a and A, ‘Check that Main compression exceeds M 0.4 x 12.2 x 1000 x 164? 2.3 x 10% Use say 0.15% area of reinforcement = 246 mm?im Ma 7.1. KNm/m > M_ ‘Ti2bars at 336 mm centres (2 x pocketspacing) gives 337 mm?/m. ‘Secondary reinforcement clause 26.3 Minimum area = 0.05% = 0.05 a 2) = T00 * 1000 x 164= 82 mm?/m Use 6mm dia. barsat 300 mm centres, (every 4th bed joint) = 94 mm?/m. Shear clauses 19.1.3.1.1 and 22.5.1 Shear force due to design loads, V = 7.8kNim 78x10 1000 x 164 ‘Therefore shear stress due to design loads,v 0.05 N/mm? Characteristic shear strength of reinforced brickwork, f, = 0.35 N/mm? (where reinforcement is surrounded with mortar) f Ym 0.17 Ninn? ‘Therefore design shear resistance of wal 4 ‘This exceeds 0.05 N/mm? and is therefore satisfactory Consider the maximum height a wall of this section may retain Design moment of resistance based on brickwork in compression = 57.1 kNm/masbefore. I M _ 571x108 Using Code Figure 3 for Q = M, = 57.110" _ 9 19, sing Code Figure 3forQ = B= Tr OO =2.12and fe 3 Nimm?,¢= 0.72 Yon Therefore A, SELL ISI 1909 mn? 460 X0.72 x 164 ‘Therefore use T16 bars at 168 mm centres (1196 mm?/m), My = 56.5 kNmim 1.4 x 0.33 x 1600 x 9.81 H? 6x 1000 a> [365K XE J Therefore H = "/74x 0.33 x 1600 x 9.1 ~ / 46-75=3.6m [Handbook to BSSea8: Par2: SECTION 2: Reinforeed Masonry 39 Design moment 5.3.3 Check shear Lateral loads, Ey = Ka pxg H?/2 0.33 x 1600 x 9.81 3.67 = ENGR 88 5 3.810 33.6 KN Shear force due to design loads, V 4x 33.6= 47.0 kNim 47.0108 Shear stress due to design loads, v = Typ < gq = 0-29 Némm’ £0.35 Ym 2.0 17 Nimm? ‘This exceeds design shear resistance of wall ‘Therefore shear is critical and H must be reduced 0.17 X 1000 % 164 _ 95.9 prj 10° / 1.4% 0.33 x 1600 x 9.81 ‘The area of reinforcement, A,, maybe reduced to suit. EXAMPLE 9, POCKET RETAINING WALL Design a reinforced brickwork cantilever pocket wall toan industrial building which is required toretaina 4.0 m height of gravel of 2000 kg/m? density. Maximum 3.0m and maximum H 2 Ney WS | so00e%6 CA L 440 Assume angle of internal friction of gravel = 35° in35 +sin35 ‘Therefore Ky 027 Lateral load onstem P = K,p,gH x H/2 0.27 x 2000 x 9.81, 4.0? _ cr x 42» a2.4knm Design lateral force, ¢P 4x 42.4 = 59.4 KNper pocket and design moment, M= 59.4 x #2 9.2 kNm per pocket ‘Trya wall width of 440 mm with pockets spaced at 1.0m to avoid the need to design the brickwork as spanning between pockets (clause 2.4.3.1). Materials ‘Assume crushing strength of bricks is35 N/mm? and use of mortar designation ii), ie from Code ‘Table3 (A), f= 9.4 Nimm? Reinforcement, f, ~ 460 N/mm? (deformed type 2) Infill concrete grade (BS 5328)30 N/mm? Partial safety factors (materials) Masonry compression, Yum = 2.3 (normal category of manufacturing control) Masonry shear, yw = 2.0 Reinforcement strength, Y= 1.15 Bond strength, Ym = 1.5, Stem design ‘Assume exposure situation E3. From Code Table 14 minimum cover = 40 mm ‘Therefore maximum bar diameter = 112.5 ~ 40 ~ 40: 32.5 mm ‘As maximum bar diameter permitted by Code clause 26.2 = 32 mm d= 440 — 40 - 16 = 384 mm say 380 mm. Design wall as flanged beam, try pocket width of 235 mm Flange depth = d/2 = 190 mm From Code clause 22.4.3.1 width of flange is least of: a)235 + 12x 190= 2515 mm, b) 1000 mm, ©) 4000/3 = 1333 mm. therefore flange width = 1000 mm 94 From Code Figure3 for fx/Ymm =3:3 = 41 M___79.2x 10° andQ = M2 8.2 x10 9) 55,¢= 0.93 = a2 1000 x 3808 therefore z = 0.93 x 380 = 353.4 mm zexceeds the minimum permitted and is therefore satisfactory Myns _ 79.2 10° x 115 fz 460 x 353.4 use 2no. T20 diameter bars per pocket (628 mm?) ‘Check that Mg based on brickwork in compression is not exceeded My==4 pre(d—0.5t) eFax 9.4 Ma=33 x10" This is greater than 79.2 kNm and the sections satisfactory. Shear clauses 19.1.3.1.2and22.5.1 Shear force due to design loads, V = 59.4 kNim % 1000 x 190 (380 ~ 0.5 x 190) = 221.3 kNm Shear stress due to design loads, 0.16 N/mm? A, 628 bd = 1000 x 380 = 9.0017 Handbook to BSS628: Part2: SECTION?: Reinforced Masonry 61 Characteristic shear strength of section, f,, = 0.35 + 17.5 x 0.0017 = 0.38 Nimm? {maybe increased bythe factor (2.5 ~0.28) 79.2 x 10° 1333 59.4 x 10° 0.31 Nimm? Thisexceeds 0.16 N/mm? and noshear reinforcement is required. NB. In his case it was unnecessary to enhance , because “3 = 0.19 Nim? > 0.16 Nimm?. Curtailment In order to avoid 4m lengths of reinforcement projecting from the foundation, the T20 bars may be curtailed at a suitable height and smaller bars lapped on, Assume T16 bars are used. In order to curtail the T20s in the tension zone, Code Clause 26.9a, b ‘orcmust be! satisfied. In this case 26.9c is appropriate. Therefore the design moment capacity of the T16s must be at least twice the design moment. Design Myof2n0. T16 bars, A, = 402mm?, assume c = 0.95 402 > 460 x 0.95 x 380 11S 10° My _ 58.0108 bd? 1000 x 3807 0.9Sis the correct assumption, M 8.0 kNm Q = 0.4, and for f,/Ymm = 4.1 Thus T20s may be terminated where the design moment, M = 29.0 kNm. O27 x 2000 x Thismoment occurs where H = V23.5 = 2.86m from top of wall Check that T20s will extend at least an effective depth or 12 diameters beyond the point where they are nolonger needed. The T16s will resist a moment of S8kNm. From above, this occurs at HeVax235<3.6m Aneffective depth above this level of wallis satisfactory. 12diameters above this level = 3.36 m therefore curtailment of T20s at 2.86 m below top of wall is satisfactory. 3.22 m, therefore curtailment of T20s at 2.86 m below top a 54 S41 Check lap length for T16. Characteristic bond strength, f, = 2.5 Nimm?, clause 19.1.6 Therefore length required to develop full anchorage bond, _ By Yn As Sims % fy Xe x Dar diameter = 4601.5 201 “T1Sx25xm% 167 00mm Thisis greater than 25 x bar diameter + 150(= 550) and therefore governs. Thus curtail T20s 1.2m above foundation and for simplicity continue T16 down to just above foundation level. Foundation design will be to BS81 10 and using standard soil mechanics procedures for bearing Pressure and stability calculations, LATERALLY LOADED MASONRY WALLS CONTAINING BED JOINT REINFORCEMENT EXAMPLE 10, LATERALLY LOADED WALL Designa 4.5 mhigh by 4.5 mlong cavity wall with a brick outer and a 100 mm thick blockwork inner leaf. The wall is reinforced with bed joint reinforcement to resist a characteristic wind load (of 0.6 kN/m*. The wall is discontinuous at its top edge and both ends, but is connected to the structure along these edges. A sheet dpc is provided at the base of the wall. Exposure situation E2. Appendix A to BS5628: Part 2 contains four alternative design methods. For comparative purposes the wall will be designed by each of these methods. Inorder to provide the maximum freedom of choice of facing brick it will be assumed that the clay brick used has a unit compressive strength of 20 N/mm and a water absorption greater than. 12%. A mortar designation (ii) isassumed. The inner leafis of 2.8 Nimm?, concrete blockwork in mortar designation (iv) Edge conditions Itisassumed thatthe supporting structure is sufficiently rigid to provide simple support tothe top and sides ofthe panel and that, because ofthe dpc, the bottom edge is also simply supported. Limiting panel dimensions Clause 23.2.4, effective thickness of the wall 22 (1025+ 10)=135 mm Clause A.2.3(b)(2) 1500 x 4500 = 20.3 x 10° mm? height x length 2400 x tor? = 2400 x 135% = 44 x 10% mm? > 20.3 x 10% mm? therefore panel size satisfactory Maximum dimension = 60 tec = 60 x 135 = 8100 mm > 4500 mm. therefore panel dimensions satisfactory Durability Clause 32.2.4 requires that where clay bricks with a water absorption greater than 10% are used the reinforcement protection should be that appropriate to the next more severe exposure situation. Therefore, from Table 13, for exposure situation E3, (instead of E2) austenitic stainless stee! reinforcement, or carbon steel coated with stainless steel, is required. 15 mm coveris required to bed joint reinforcement, Incach design method itis necessary to establish the strength of the panel in accordance with 1BS5628: Part 1 asifit was unreinforced. Capacity of unreinforced wall BS5628 : Part | Clause 36.4 Outer leaf ux perpendicular = 0.9 Nimm?, fx, parallel 0.3 Nimm?,¥q 5.1 = 0.90 1.2, Handbook to BSS628: Part? SECTION?: Reinforced Masonry 6 = 82.75 x 108 mm 2.25 x 1000 Self weightstress at mig height 2021S 1 5 XY = 0340.14 .04 x 0.9 x 3.5 0.49 Therefore p= 09 Inner leaf ex perpendicular = 0.4N/mm? fy parallel = Z= 1.6710 mm/m )2Nimm?, Ym = 3.5, ¥¢= 0.9 0F 1.2 ; seit = 0.8% 2.25 x 1000 Self weight stress at mid height = 9-3 22°70 ).06 Nim? XX Ym = 0.018 X 0.9 X3.5 ‘Therefore w= "=F = 0.65 Design mament of resistance outerleaf a x @) From Table 9E, « (outer leaf) = 0.057 anda (inner leaf) = 0.051 Ifcharacteristic load on outerleaf = Wo and characteristicload on inner leaf = W: then, design moment outerleaf ).057 x Wo X 1.2.x 4.5?= 1.39 WokNm/m @) Design moment innerleaf 051 x Wy x 1.2 4.5? = 1.24W, KNmvm - 4) Equating (1) & (3) and solving for Wo Wo =0.32 kNim? Equating (2) & (4) and solving for Wi Wy = 0.15 Nim? ‘Therefore characteristic load capacity for unreinforced wall = 0.47 Nim? ‘Therefore unreinforced wall isinadequate. 1BS5628 : PART 2, APPENDIX A DESIGN METHODS ‘A.3Method One ‘Assume only brick leaf reinforced Maximum permitted enhancement of load capacity over that for the unreinforced wall panel = 50%. Reinforce outer leaf with prefabricated stainless steel parallel wire lattice reinforcement. 4.0mm diameter .! ‘60mm aa Eat eaeeer stainless steel, f, = 485 Nimm? | length of reinforcement = 3.0 m | = | | _ provide reinforcement with laps 1 | staggered as shown | on78 | Cl 2 2 Design moment = Ws? _ 1.2% 0.6% 4-5? 1 99 mim 8 8 M__1.82x 108 = MRI 20.32 Ob” 1000752 {i from Table 2 (a), BS5628 : Part 1, = 5.8N/mm? (NB: Part I must be used to obtain values for masonry built with mortar designation (iti). fe 5.8 . Therefore * = 3:5 = 1.66 (ym from BSS628: Part 1). Therefore, from Figure 3, BS5628 : Part 2, = 0.89 ).89 x 75 = 66.7 mm andz= ‘Therefore area of reinforcement required, 65 mm?im therefore satisfactory ‘Minimum area of reinforcement, Clause A.2.4, 14 mm? at not greater than 450mm centres = 31 mm*im, therefore A, issatisfactory (Check that compressive strength of brick is adequate. Clause 22.4.2.1 0.4 x 5.8 x 1000 x 75? 3.5 x 10° Inorder to avoid excessive deflection or cracking (ic. serviceability failure) in the wall panel, the enhancement of lateral load resistance over the equivalent unreinforced wall panel is limited 050%. Because, in this example, only the outer leaf is reinforced, the enhanced resistance is compared with the resistance of the unreinforced outer leaf alone: reinforced resistance on uunreinforced resistance ~ 0.33 ~ | S8i.¢. 88% Me = 3.7kNm/m therefore satisfactory As thisis greater than 50%, either both leaves must be reinforced or a serviceability check can be carried out on the outer leaf, although such a check is not referred to in the Code. [tis however considered to bea reasonable approach. The Appendix A design methods are based on the premise, given in A.2.1, that the crack load for a bed joint reinforced, laterally loaded, wall is comparable with the ultimate load for the equivalent unreinforced wall. From page 64 the oad (Ultimate Limit State) for the unreinforced outer leaf is 0.32 kNim?. ‘Therefore the ultimate load (ie failure strength), with y; = 1.0 andy = 1.0, of the outer leaf = 1253.5 2. £0.32 755 9 = 134 kN’ Asthisis at least as great as the cracking load for the equivalent reinforced leaf, the serviceability design load capacity of the reinforced leaf 134 _ 134 A Foe TS 7 O-9KNIE (Ym from 20.3.2) ‘This exceeds the serviceability design load on the wall of 1 X 0.6 = 1.0 X 0.6 = 0.6kNim? and. cracking will not occur. The serviceability limit state of deflection should also be considered and ‘this will be as discussed in A.6 Method Four on page 67. Itcan be scen that the enhancement limit given in the Code can be exceeded, provided that a Serviceability Limit State design check iscarried out. A.4Method Two Assume only brick leaf reinforced. ‘This method is imilar to Method One except that the reinforced section is only required to resist the excess load above that carried by the unreinforced masonry. Iti therefore, less conservative than Method One and the enhancements limited to 30%. Load capacity of unreinfoyced leaf = 0.32 kN/m?, see page 64 Excess load = 0.6 ~ 0.32 = 0.28 kNim?>0.3 x 0.32 ‘Therefore, either both leaves must be reinforced or a serviceability check carried outon the outer leaf as for Method One. Use same type of reinforcement as in Method One,d=75 mm 0,28 x 4.5? Design moment 2 Wi? 85 kNm/m M__ 0.85 x 10° bd? ~ 1000 x75? a= fe Ym ‘Therefore, from Figure 3, BS5628 : Part 2, c= 0.95 (maximum) andz=0.95 x 75=71.2 mm 0.85 x 10° 1.1 485 71.2, ‘Minimum area of reinforcement = 31 mm?/m (as for Method One) =0.15 1.66as for Method One 28.3 mm?/m ‘Therefore, provide reinforcement every 4th course (300 mm) Ay=41 mm?/m Stagger laps.as for Method One. Serviceability check Asthe serviceability design load capacity of the reinforced leaf is considered to be independent, of the area of reinforcement for commonly used percentages of bed joint reinforcement, clause A.2.1, the serviceability check required here is the same as for Method One. A.5 Method Three Assume only brick leaf reinforced. Because of the differencesin load/deflection behaviour between the reinforced and unreinforced leaves, where only one leafis reinforced, the reinforced leaf should be designed to resist the full load. ‘Therefore, required design capacity of outer leaf = 0.6 x 1.2=0.72 kNim? ‘Try reinforcement every second course Therefore A, = 84 mm?/m Assuming lever arm factor, c= 0.95, givesz = 71.2 mm 84x 485 x 71.2. 1.135 x 10° Mg, for bending about the horizontal axis (unreinforced), (0.3 +0.14) 1.75 x10" _ 9 99 Navin 3.5% 10 ” and Ma 2-52. kNmm/m for bending about the vertical axis. ‘Therefore, orthogonal ratio, From Table 9, BS5628 : Part 1 value of « for minimum p = 0.3 is 0.068, however, where the horizontal span strength is much greater than the vertical span strength, the value of a will tend 100.125. Use this value. ‘Therefore, design moment, M = aye WyL? = 0.125 x 1.2 0.6 x 4.5?= 1.82 kNm/m This s less than 2.52 kNm/m and, therefore, satisfactory. ‘Stagger laps as for Method One and check that bond stress at lap position is satisfactory, .088 Using yield line theory to calculate the value of a, an orthogonal ratio of 0.09 gives« and thusa smaller design moment. However this is outside the range of usual orthogonal ratios and has no empirical justification and the use of a = 0.125is therefore considered appropriate. ‘The Code does not, at present, limit the load capacity enhancement over the equivalent unreinforced wall. This is somewhat illogical since in other respects the design issimilai to Method One and, therefore, a maximum enhancement of 50% may be considered appropriate. ‘A.6 Method Four Inthis approach the failure strength of the unreinforced wall is taken to be the cracking strength ofthe reinforced wall Considering both leaves, the characteristic load capacity of the unreinforced wall = 0.47 kN/m?, see page 64, calculated using ym = 3.5.and yy = 1.2 1.0and y= 1.0, ultimate load (ie failure strength) L.97kN/m? ‘Therefore with», ‘Thus the serviceability design load capacity (cracking) 1.97 _ 1.97 So2 =F = LBL kW Co from 20.3.2) ‘This, therefore, is the design wind load capacity for the serviceability limit state of cract (ie = 1.0). Itexceeds 0.6 kN/m? ands therefore satisfactory. Using Method Four, the wall is capable of resisting a greater wind load than using Methods One, ‘Twoand Three when the enhancement limits are applied. However the wall must be reinforced toensure thatin resisting the design serviceability load, the ultimate limit state isnot reached. Toachieve its full serviceability design’ resist an excess ultimate design load of: 1.2(1.31-0.47) reinforced), However, for the situation in the example, the wall has only to resist a characteristic wind load of 0.6kNim*. Asthe reinforced walls to be designed in accordance with Method Two. itcan be seen from page 66 that reinforcement in every fourth bed joint (300 mm) of the outer leaf will provide the necessary design resistance. As only one leafs reinforced the serviceability checks, ‘must be carried out on that leaf alone; again this sas for Method Two (page 66). 1BS5628: Part2 recommends that with this design method the serviceability limit state of ad capacity the wall would have tobe reinforced to 01 kN/m*in accordance Method Two (assuming that both leaves are Handbook to BS3628: Part2: SECTION2: Reinforced Masonry 67 deflection should be checked. This snot straightforward. Itis recommended that elastic plate theory, assuming that the masonry is unreinforced and using the values for short term elastic moduli given in Appendix Cto the Code. In this case for simplicity, the panel will be considered tospan horizontally. Ifthe design load (serviceability) is considered to be carried by the outer leaf: x 0:6. 4.5* x 10 eae deflection = 354 900 x5.8 x 8.97 X10? ‘Thisis less thari the limit of span 250 (= 18mm) recommended in clause 16,2.2.1 and therefore satisfactory. Ifthe risk of damage to finishes was critical and a lower deflection limit set, it ‘would be necessary to carry out a more rigorous analysis eg. using elastic plate theory. Ifboth leaves are reinforced it will be necessary to carry out a serviceability check on both leaves. It is suggested that, in order to do this, the load is apportioned between the leaves on the assumption that both leaves deflect equally. This apportionment can be done by equating the deflection equations for each leaf and solving for the individual leaf loads from knowledge of the total design serviceability load on the wall. These individual leaf loads can then be compared with their serviceability load capacities as in Method Two on page 66. Each leaf can then be reinforced (using Method Two) to resist their respective Ultimate Limit State design loads. Itcan be seen that Method Four is effectively the same as Method Two when the proposed serviceability design checks are carried out inthe latter. As has also been seen, because of the suggested limitations of the extrapolation of bending moment coefficients for iow orthogonal ratios, Methods One and Three often effectively become the same. Methods One and Three are more conservative than Two and Four, since the latter make allowance for the load capacity of the unreinforced masonry. There is, thus, scope for the clarification of these design methods, anditis likely that this will be dealt with in a future amendment to BS 5628: Part 2. 6.0 REFERENCES 1. British Standards Institution. Use of Masonry. Part 1. Structural use of unreinforced masonry. BS 5628: Part 1: 1978 (1985). Part2. Structural use of reinforced and prestressed masonry. BS 5628: Part 2: 1985. Part 3. Materials and components, design and workmanship. BS 5628:Part 3: 1985. 2. British Standards Institution. Structural Use of Concrete. Part 1. Code of Practice for Design ‘and Construction. BS8110: Part 1: 1985. h Standards Institution. The Structural Use of Concrete. CP110 : 1972 (now superseded 4, Civil Engineering Code of Practice No. 2 (1951). Barth Retaining Structur 5. Terzaghi and Peck, Soil Mechanics in Engineering Practice, Wiley Internati 6. Brickwork Dimensions Tables, BDA Design Note 3 July 1979. 7. Haseltine and Tutt, Brickwork Retaining Walls, BDA Design Guide No. 2, 1991. APPENDIX A PROPOSED AMENDMENTS 10 BS5628: PART 2 Anumber of errors or anomalies in BSS628: Part 2 have been referred to in the text of this, Handbook. The most significant of these are included in a draft revision to the Code at present being prepared for publication. The major revisions are noted below. CLAUSE 19.1.4, TABLE4 Inline with revisions to other related British Standards, Table 4 will be amended as shown below: TABLE4. CHARACTERISTIC TENSILE STRENGTH OF REINFORCING STEEL, fy Designation Grade Nominal CCharactertte se tensile strengthty Hot rolled pain steel bars complying with 250 All 250 BS4449 Hot rolled and cold worked deformed bars 40 all 0 complying with BS#49 Cold reduced steel wire complying with BS 4482 e Uptoand ca used in stelfabicin accordance with BS4483 Including 12mm ‘Types304 and 316plain stainlesssteel bars 250 all 250 complying with BST ‘Types 304 and 316deformedstainless steel bars 460 all 0 complying with BS6744 CLAUSES 23 AND 24 ‘These clauses make numerous references to ‘axial’ loads, where eccentric vertical loading is being referred to. In order toclarify the meaning itis proposed that the word ‘vertical’ will be substituted for the word ‘axial’ Insub-clause 23.3.1.2in the expression for My the term ) should be inverted to read (%). ‘The expression for Nycis also incorrect, it should be:— Nae = fic Arm, Where the meaning of the termsis unchanged. Associated with both of the above expressionsis Table 12 which gives values for the coefficient a. Itis proposed to alter this table slightly, as shown below: TABLE 12 VALUES OF THE COEFFICIENT « eat ° 1.00 on 088 02 on 03 0.65 _ oa 058 _ 05 02 206 030 Handbook to BSS628: Part? SECTION2: Reinforced Masonry o APPENDIXB (COMMON BOND PATTERNS FOR REINFORCED MASONRY T 4 pocket bao! et wall english bond: alternate courses dotted \L waltes to 8S5628: pant 2, Appendix 3 routed cavity wall ‘THE BDA DESIGN GUIDESERIES ‘The BDA Design Guide series of publications provides a range of titles covering many aspects ofboth unreinforced and reinforced structural use of brickwork masonry. Other publications in the series relevant to reinforced brickwork design are: = The Design of Brickwork Retaining Walls; Design Guide 2 = Designing in Reinforced Brickwork; Design Guide 14 ‘= Handbook to BS 5628:Part 2: Section 1:Reinforced Masonry; Design Guide 17 = Design of Post Tensioned Brickwork; Design Guide 20 ‘These titles and the full publi Department, ions listing are available from BDA’s Publications Sales Tank Walter. Architects - Porter Wright, Structural engincers—David Goodwin Associates. Inside fron cover: Photography John Adams. Architects and structural engineers Oxforushire County Surveyor & Engincer in collaboraiion with Bradshaw Buckton & Tange, and the Bitish Ceramic Association. Paged Photography - BDA, Architects -Green Lloyd, Structural engineers Fothergill& Co, Page 6 Photography ~ Graham Jennings. Architects- The Salvation Army International Headquarters. Structural engineers Curtns. Page: Photography Frank Walter. Architects Covel Matthews Wheatley, Structural engineers ~ Deakin Callard & Parincrs. Page 22: Photographylohn Adams, Architect -F, R, Wallers, District Architect at Mans engineers—Curtns. Photographs on pages 7, 18,24,27,31, 32, Sand 72 were provided by Stuart Bell Director, Marshalls Clay Produets Lt (© The Brick Development sociation, Woodside House, WinkMeld, Windsor, Berks SL42DX. Tel: O34 B8S651 “Thecontentsof this publication are intended for general guidance onlyand any person intending touse these contents forthe purposes of design. consiuetion or repair of brickwork or any related projec should first consulta Professional Advisor The Brick Development Associaton its servants and any persons who contributed to or whose in anyway connected with his publication accept no liability arising from negligence or otherwise howsoever cauted for any injury or damage io any property asa result of any oF Feliance en any method product instuction, or other contents ofthis publication Designed and produced forthe Brick Development Assocation, Woodside House, Winkfield, WINDSOR BerkshireSL42DX. ‘Telephone Winkficld Row (i344) 885581 by David Goad MSTD. Techneaillustration: ohn James, Printed: Prima Print Handbook to BSS428: Part2: SECTION2: Reinforced Masonry 7

Potrebbero piacerti anche